Benign gynae Flashcards

1
Q

Fibroids: All of the following statements are true except:

a - >20% of fibroids have a chromosomal abnormality
b - on cytogenetics a single fibroid comes from one single cell (not pleomorphic)
c - 20% of women develop a fibroid
d - MPA decreased mitotic activity in fibroids

A

a - >20% of fibroids have a chromosomal abnormality

MPA - medroxyprogesterone acetate

How well did you know this?
1
Not at all
2
3
4
5
Perfectly
2
Q

Red degeneration of a fibroid:

a- causes an elevation of the ESR
b - causes leucopaenia
c - only occurs in pregnancy
d- occurs due to embolisation of the feeding vessels

A

a- causes an elevation of the ESR

How well did you know this?
1
Not at all
2
3
4
5
Perfectly
3
Q

Regarding dermoid cysts:

A - the most common ovarian mass in pregnancy
B - 10% are malignant
C - 40-50% are bilateral
D - oophorectomy required for adequate removal

A

A - the most common ovarian mass in pregnancy

How well did you know this?
1
Not at all
2
3
4
5
Perfectly
4
Q

A 15 yr old girl presents with pelvic pain and an US shows a 4 cm ovarian cyst. What is the commonest cause?

A - Dermoid cyst
B - Follicular cyst
C - Corpus luteal cyst 
D - Serous adenoma
E - Endometrioma
A

C - Corpus luteal cyst

Range in size from 2-5cm

How well did you know this?
1
Not at all
2
3
4
5
Perfectly
5
Q

A woman with PCO. LMP 25 days ago. Usually has 3 periods a year. Heavy flow day 10-12. Endometrium is likely to show?

A - subnuclear vascuolisation
B - haemorrhagic and necrotic glands
C - crowding of straight glands

A

C - crowding of straight glands

How well did you know this?
1
Not at all
2
3
4
5
Perfectly
6
Q

What is the rate of amenorrhoea in patients with intrauterine synechiae?

A - 10%
B - 20%
C - 40%
D - 60%
E - 80%
A

C - 40%

As per UpToDate

Can be caused by TB
Rx: hysteroscopic resection

How well did you know this?
1
Not at all
2
3
4
5
Perfectly
7
Q

What is the most common symptom of benign breast disease?

A - pain 
B - tender lump
C - change in breast size
D - discharge
E - change in menses
A

B - tender lump

How well did you know this?
1
Not at all
2
3
4
5
Perfectly
8
Q

Advantages of GnRH agonist for the treatment of fibroids include all of the following EXCEPT:

A - allow vaginal hysterectomy
B - allow return of patient Hb towards normal before surgery
C - diagnostic test to distinguish between fibroid and leiomyosarcoma
D - allows hysteroscopic resection of fibroid
E - reduced intraoperative blood loss

A

C - diagnostic test to distinguish between fibroid and leiomyosarcoma

How well did you know this?
1
Not at all
2
3
4
5
Perfectly
9
Q

What percentage of fibroids will shrink with GnRH analogues?

A - 10%
B - 25%
C - 50%
D - 75%
E - 90%
A

C - 50%

How well did you know this?
1
Not at all
2
3
4
5
Perfectly
10
Q

After 6 months of GnRH analogues, how much reduction in uterine size would you expect?

A - 10%
B - 25%
C - 50%
D - 75%
E - 90%
A

C - 50%

How well did you know this?
1
Not at all
2
3
4
5
Perfectly
11
Q

The commonest cause of dyspareunia is

a- inadequate lubrication
b - thrush
c- psychosexual issues

A

a- inadequate lubrication

How well did you know this?
1
Not at all
2
3
4
5
Perfectly
12
Q

30 yo para 1 with a 3 yo child presents with menorrhagia and on VE there is a 12 week fibroid uterus palpable. She would like another child in the future. Management:

a - TAH
b - Myomectomy abdominally
c - GnRH analogue for 6/12
d - Continuous provera for 9/12
e - Hysteroscopic resection
A

d - Continuous provera for 9/12

Symptomatic management initially

How well did you know this?
1
Not at all
2
3
4
5
Perfectly
13
Q

Which of the following is correct in regard to premenstrual syndrome?

A - it is due to low progesterone level
B - bromocriptine is more effective than cyclical synthetic progesterone in treating PMS
C - cyclical progesterone showed no advantage over placebo in treating PMS
D - it is due directly to endogenous endorphin withdrawal
E - it is related to HLA B27 typing

A

C - cyclical progesterone showed no advantage over placebo in treating PMS

How well did you know this?
1
Not at all
2
3
4
5
Perfectly
14
Q

A woman had a NVD with an episiotomy and has been BF for 8 weeks. She presented complaining of dyspareunia. O/E the episiotomy is well healed. What is the most likely diagnosis?

A - suture granuloma
B - atrophic vaginitis
C - PND
D - Narrowed introitus

A

B - atrophic vaginitis

How well did you know this?
1
Not at all
2
3
4
5
Perfectly
15
Q

Midcycle spotting
A - E breakthrough
B - E withdrawal

A

B - E withdrawal

How well did you know this?
1
Not at all
2
3
4
5
Perfectly
16
Q

A 46 yo lady presented with severe menorrhagia for six months and clinical evidence of a tender enlarged uterus. What is your first investigation?

A - hysteroscopy and D&C
B - FBC
C - LH and FSH
D - Coagulation profile
E - Serum progesterone
A

B - FBC

How well did you know this?
1
Not at all
2
3
4
5
Perfectly
17
Q

A woman presented to you with lack of sexual excitement. What is the most likely reason?

A - fear of pregnancy
B - poor coital techniques
C - marital discordance
D - endometriosis
E - pelvic congestion syndrome
A

C - marital discordance

How well did you know this?
1
Not at all
2
3
4
5
Perfectly
18
Q

25 yo. O/E 5 cm simple cystic R adnexal mass confirmed on US. Mx?

A - repeat exam in 3/12
B - give OCP and repeat US in 1/12
C - laparotomy
D - laparoscopy and aspiration of cyst

A

A - repeat exam in 3/12

<50mm diameter - do not require f/u
- Likely physiological and almost always resolved within
50-70mm - yearly USS f/u

How well did you know this?
1
Not at all
2
3
4
5
Perfectly
19
Q
25 yo, 1 yr Hx acute virilizing symptoms
LH 2	
FSH 3	
Testosterone 2-3x normal	
DHEAS normal	
17 OHP Normal
A - Cushing’s syndrome
B - PCO
C - Late onset CAH
D - Sertoli-leydig tumour
E - Adrenal cortical adenoma
A

D - Sertoli-leydig tumour

How well did you know this?
1
Not at all
2
3
4
5
Perfectly
20
Q
43 yo, recent onset virilizing symptoms
LH 1	
FSH 1	
Test 2x normal	
DHEAS 1.5x normal	
17 HOP normal
A - Cushing’s syndrome
B - PCO
C - Late onset CAH
D - Sertoli-leydig tumour
E - Adrenal cortical adenoma
A

E - Adrenal cortical adenoma

How well did you know this?
1
Not at all
2
3
4
5
Perfectly
21
Q
22 yo, long Hx irregular menses and hirsuitism
LH 12	
FSH 6 	
Test 1.5x normal	
DHEAS 1.5x normal	
17 HOP 2x normal
A - Cushing’s syndrome
B - PCO
C - Late onset CAH
D - Sertoli-leydig tumour
E - Adrenal cortical adenoma
A

C - Late onset CAH

How well did you know this?
1
Not at all
2
3
4
5
Perfectly
22
Q
22 yo long Hx irregular menses and hirsuitism
LH 1.9	
FSH 6	
Test 1.5x normal	
DHEAS 1.5x normal	
17 HOP normal
A - Cushing’s syndrome
B - PCO
C - Late onset CAH
D - Sertoli-leydig tumour
E - Adrenal cortical adenoma
A

B - PCO

How well did you know this?
1
Not at all
2
3
4
5
Perfectly
23
Q

47 yo has progressive menorrhagia with regular cycles. O/E uterus normal anteverted with no adnexal masses. Hysteroscopy – regular cavity, no pathology.
D&C – secretory normal endometrium.
Mx:

A - cyclic Progesterone
B - OCP
C - NSAIDS
D - Advise endoablation, compared to TAH is more effective and less complications
E - Advise endoablation is adequate contraception

A

A - cyclic Progesterone

24
Q

How long does it take to stop heavy bleeding with GnRH analogues?

A - 12 hrs
B - 18 hrs
C - 1 week
D - 4 weeks

A

D - 4 weeks

Begins to work after 2 weeks

25
Q

A woman has a regular period every 28 days. She has spotting around day 14 of her cycle. Which of the following is correct?

A - oestrogen BTB
B - progesterone BTB
C - oestrogen withdrawal bleeding
D - progesterone withdrawal bleed

A

C - oestrogen withdrawal bleeding

Fall in estrogen levels just before ovulation –> spotting

26
Q

Fibroids. Which is true?

A - generally cause pain
B - increased in nulliparous women
C - 1% become sarcomatous

A

B - increased in nulliparous women

27
Q

A 35 yo with menorrhagia and a 6 cm intramural fibroid wishes to become pregnant. Should she?

A - ignore fibroid and attempt to conceive
B - 3/12 of GnRH agonist then attempt to conceive
C - undergo myomectomy at laparotomy after GnRH analogue
D - undergo hysteroscopic resection of fibroid

A

A - ignore fibroid and attempt to conceive

There is insufficient evidence to determine whether myomectomy for IM fibroids improves fertility outcomes
>8cm associated with infertility, <5cm not major role

28
Q

You have been asked to see a 22yo nulligravida who has oligomenorrhoea and idiopathic hyperprolactinemia. She desires pregnancy. Her physician initiated 2.5mg bromocriptine BD, and she is experiencing orthostatic symptoms and moderate nausea. Serum bHCG is negative, TSH normal and MRI normal. The most appropriate next step in her management is to

a. Advise her to continue bromocriptine and she will eventually become tolerant of the medication
b. Discontinue the bromocriptine
c. Reduce the dose to ½ tablet at bedtime until she becomes tolerant of the medication
d. Tell her to take an extra dose at bedtime

A

c. Reduce the dose to ½ tablet at bedtime until she becomes tolerant of the medication
1. 25mg nocte for 5 nights, and gradually up titrate to 7.5mg daily in 2-3 divided doses over about 3 weeks

Common adverse effects:

  • N/v, headache, postural hypotension, vertigo, GI disturbance
  • Minimised by taking at night and then taking tablets with food
29
Q
A 22yo woman has had severe hirsutism for 7 years and oligomenorrhoea since menarche. Her sister has mild hirsutism and uses the COCP. She is 1.5 metres tall and weighs 50kg. Pelvic examination shows borderline clitoromegaly but is otherwise normal. Hormone profile shows
LH	19	[5-25]
FSH	6	[4-22]
Testosterone	3	[0.5-2.6]
DHEAS	21	[0.9-11.7]
17-OH progesterone	16	[0.8-8.0]

The MOST LIKELY diagnosis is:

a. PCOS
b. Ovarian hyperthecosis
c. Late onset CAH
d. Androgen secreting adrenal tumour
e. Androgen secreting ovarian tumour

A

c. Late onset CAH

30
Q

Which of the following drugs is NOT associated with non-androgen dependent hair growth (hypertrichosis)

a. Phenytoin
b. Cyclosporin A
c. Ranitidine
d. Diazoxide

A

c. Ranitidine

31
Q

Which of the following is true concerning uterine fibroids

a. They undergo sarcomatous change in 1% of cases
b. They are associated with nulliparity
c. They characteristically cause pain
d. They should be removed at Caesarean if larger than 2cm diameter

A

b. They are associated with nulliparity

32
Q

All are characteristic behavioural components of the Chronic pelvic pain syndrome EXCEPT

a. Pain is refractory to medical management
b. Signs of depression have begun
c. The patients role in the family has changed
d. A history of sexual abuse is usually present

A

d. A history of sexual abuse is usually present

33
Q
  1. In the HAIR-AN syndrome, the mechanism of insulin resistance is

a. Abnormalities in the insulin receptor
b. Obesity related changes in ovarian function
c. Underlying diabetes mellitus
d. Alterations in adrenal steroid synthesis pathways
e. Alterations in ovarian steroidogenic enzymes

A

a. Abnormalities in the insulin receptor

Hyperandrogenism
Insulin resistance
Acanthosis nigricans

34
Q
  1. The most common cause of introital dyspareunia is:

a. Monoilial vulvovaginitis
b. Herpes genitalis
c. Inadequate arousal
d. Vaginismus

A

c. Inadequate arousal

35
Q
  1. A blood assay of which of the following is the most direct measure of adrenal androgen activity

a. Androstenedione
b. Cortisol
c. DHEAS
d. Testosterone

A

c. DHEAS

36
Q

A 25yo presents with virilisation over the past year. Her menstrual cycles are irregular and infrequent. On examination she is obese, has marked facial hair growth and clitoromegaly. Uterus is slightly enlarged, ovaries not easily palpated. Hormone profile shows testosterone of 8 [normal 1-4], and DHEAS of 8 [normal 1-10]. The MOST APPROPRIATE next step in diagnosis is:

a. Vaginal ultrasound of the ovaries
b. Dexamethasone suppression test
c. Measurement of serum 17-OH progesterone
d. Measurement of serum androstenedione

A

a. Vaginal ultrasound of the ovaries

37
Q
  1. Regarding therapy for mild to moderate endometriosis, which of the statements below is LEAST CORRECT

a. Monthly fecundity over the first 36 weeks is significantly increased after laparoscopic treatment of mild or moderate endometriosis
b. There is a tendency for fertility to plateau at the same level with either surgery or expectant management
c. Danazol reduces both pain and disease progression
d. After a course of GnRH analogue therapy, fecundity is transiently improved in comparison to expectant management

A

d. After a course of GnRH analogue therapy, fecundity is transiently improved in comparison to expectant management

38
Q
  1. The principle secretory product of polycystic ovaries is

a. Androstenedione
b. Testosterone
c. DHEAS
d. Oestrone
e. Oestradiol

A

a. Androstenedione

39
Q
  1. In patients with hyperandrogenic chronic anovulation, all of the following are described EXCEPT

a. Raised LH:FSH ratio in serum
b. Hyperoestrogenism
c. Raised serum triglyceride, reduced HDL cholesterol
d. Increased SHBG

A

d. Increased SHBG

Insulin resistance –> increased insulin –> increased androgen production by ovarian theca cells and reduced hepatic synthesis of sex hormone-binding globulin

Low SHBG –> high levels of free testosterone

40
Q
  1. In patients with hyperandrogenic chronic anovulation, which is LEAST CORRECT?

a. Insulin inhibits granulosa cell production of IGF-1 binding protein
b. Insulin inhibits hepatic production of SHBG
c. Insulin, through homology with IGF-1, binds to the IGF-1 receptor
d. Metformin increases circulating androgen levels by reducing serum insulin

A

d. Metformin increases circulating androgen levels by reducing serum insulin

41
Q
  1. Amenorrhoea and galactorrhoea is LEAST likely to be caused by

a. Pituitary tumour
b. Chlorpomazine
c. Thyrotoxicosis
d. Metoclopramide

A

c. Thyrotoxicosis

Hypothyroidism more common to cause galactorrhea and amenorrhoea

Hyperthyroid rarely can

42
Q
  1. Which of the following statements regarding the mechanism of anti-androgen effects is LEAST correct

a. Cyproterone acetate inhibits 5-alpha reductase
b. Spironolactone competitively inhibits binding to the dihydrotestosterone receptor and inhibits 5-alpha reductase
c. Finasteride inhibits 5-alpha reductase
d. Cimetidine weakly binds to the dihydrotestosterone receptor

A

a. Cyproterone acetate inhibits 5-alpha reductase

Anti-androgen progesterone
Ginet

Finasteride used for male pattern baldness

43
Q
  1. A 22yo woman has irregular, heavy menses a few times a year, lasting 10 days, and hirsutism. LMP was 25 days ago. Endometrial biopsy would show:

a. Supranuclear vacuoles
b. Stromal oedema with perivascular decidualisation
c. Haemorrhagic stroma and collapsed endometrial glands
d. Crowding of straight tubular glands

A

d. Crowding of straight tubular glands

44
Q

A 75yo woman is referred with generalised lower abdominal pain, loose stools, and fever of 2 days duration. She is not sexually active and there is no vaginal discharge. She has urinary urgency, but midstream urinalysis is negative. The most likely cause of her symptoms is

a. Pelvic inflammatory disease
b. Endometrial cancer
c. Urinary tract infection
d. Diverticulitis

A

d. Diverticulitis

45
Q

In obese postmenopausal women, peripheral conversion of oestrogen precursors results primarily in the formation of

a. Oestrone
b. Oestradiol
c. Oestriol
d. Androstenedione

A

a. Oestrone

46
Q

A previously normotensive 42yo woman is found to have a diastolic BP of 95mmHg at her annual examination. You should

a. Recheck BP in 2 weeks
b. Recheck BP in one year
c. Commence a thiazide diuretic
d. Commence an ACE inhibitor

A

a. Recheck BP in 2 weeks

47
Q

A major histologic ovarian structural change associated with menopause is proliferation of

a. Theca interna cells
b. Granulosa cells
c. Stromal cells
d. Epithelial surface cells

A

c. Stromal cells

48
Q

Which of the following is LEAST likely to help with hot flushes

a. Venlafaxine
b. Fluoxetine
c. Clonidine
d. Phytoestrogens

A

d. Phytoestrogens

Phytoestrogens

  • Weak oestrogens found in plants
  • not shown to reduce VMS
49
Q

A slim 58yo who experienced menopause 6 years ago and has taken intermittent oestrogen replacement until 6 months ago, consults you for HRT. She has debilitating hot flushes. She has a negative family history and doesn’t smoke. Physical and pelvic examination reveals a pale vagina, small uterus and cervix. You counsel her that the BEST hormone replacement is

a. Cyclic CEE 0.625mg days 1-25, and MPA 10mg days 16-25
b. Sequential CEE 0.625mg daily, and MPA 10mg days 21-30
c. CEE 0.625mg and MPA 5mg daily
d. CEE 0.625mg days 1-25, MPA 10mg days 13-25

A

c. CEE 0.625mg and MPA 5mg daily

Use continuous is >12 months amenorrhoea or >12 months of cyclical HRT done

50
Q

A 30yo has received conflicting advice about HRT. Which would be the CORRECT information to give this patient

a. Bone loss is most rapid in the first 3 years after menopause
b. Ingestion of 1500mg elemental calcium daily will lead to an increased incidence of kidney stones
c. HRT is not indicated above age 60
d. HRT increases the incidence of coronary heart disease
e. HRT increases the incidence of colorectal cancer

A

a. Bone loss is most rapid in the first 3 years after menopause

Average woman loses up to 10% of her bone mass in the first 5y after menopause

51
Q

Which is LEAST CORRECT of simple ovarian cysts in postmenopausal women

a. Aspiration is associated with a recurrence of 25% at 3 months
b. Approximately 20% resolve over the next 2-3 months
c. Laparoscopic BSO is the usual procedure of choice if surgery is to be undertaken
d. Less than 1 percent of such cysts are malignant

A

c. Laparoscopic BSO is the usual procedure of choice if surgery is to be undertaken

??? not sure if this is correct

52
Q

The most common benign tumour of the ovary in a postmenopausal woman is

a. Cystic teratoma
b. Serous cystadenoma
c. Mucinous tumour
d. Endometrioid tumour

A

b. Serous cystadenoma

53
Q

In recommending influenza vaccine to your patients, all of the following are correct statements EXCEPT

a. 90% of influenza deaths occur in individuals over age 60
b. The incidence of clinical and serologic influenza is reduced by 50% with immunisation
c. Advanced cardiac or pulmonary disease is a contraindication to vaccination
d. The pulmonary complications of influenza are reduced by 70% in immunised populations.

A

c. Advanced cardiac or pulmonary disease is a contraindication to vaccination

54
Q

Which of the following is LEAST CORRECT with regard to the Womens Health Initiative trial of HRT

a. Fractures are decreased with HRT
b. The trial included a substantial proportion of women with risk factors for arteriosclerosis, including hypertension, age > 70, and diabetes
c. Total cancer incidence is similar in HRT and no HRT groups
d. HRT increased the incidence of myocardial infarctions and CVA, even after adjusting for risk factors for arteriosclerosis.

A

d. HRT increased the incidence of myocardial infarctions and CVA, even after adjusting for risk factors for arteriosclerosis.

55
Q

Which is FALSE regarding raloxifene

a. It has been shown to increase bone mineral density in spine and hip
b. It reduces fracture rates in osteoporosis
c. It does not produce endometrial hyperplasia
d. Oestrogen but not raloxifene has been shown to reduce atherosclerosis in primate studies

A

d. Oestrogen but not raloxifene has been shown to reduce atherosclerosis in primate studies

EVISTA is indicated for the prevention and treatment of osteoporosis in post-menopausal women